Help pleaseee i am awful at math

Help Pleaseee I Am Awful At Math

Answers

Answer 1

Step-by-step explanation:

use the above process and answer is $3.60

Help Pleaseee I Am Awful At Math

Related Questions

A different rectangle has a diagonal of 60 yards and a height of 40 yards.find the width of the rectangle rounded to the nearest hundredth

Answers

Answer:

2159.6 i think

Step-by-step explanation:

i used a rectangle calculator

Find the volume of a cone with a base diameter of 12 m and a height of 11 m.

Answers

Step-by-step explanation:

the volume of a cone is

ground area × height / 3

and the ground area is a circle.

so,

pi×r²×h/3

with r being the radius (half the diameter) and h being the height.

therefore, we get

pi×(12/2)²×11/3 = pi×6²×11/3 = pi×12×11 = 132pi =

= 414.6902303... m³

A manager drew this box-and-whisker plot to represent the number of minutes each of the company's 360 employees took on their break.

How many employees took a break that lasted between 37 and 41 minutes?

Select from the drop-down menu to correctly complete the statement.

Answers

Given the box-and-whisker plot, the number of employees that took a break that lasted between 37 - 41 minutes is: 180.

What is a Box-and-whisker Plot?

A box-and-whisker plot is a graphical representation of a data set showing how the data set is distributed into percentiles such as 25%, 50%, and 75%.

The range of the rectangular box covers 50% of the data set.

Thus, 37 - 41 minutes lies between 50% on the box-and-whisker plot, therefore:

50/100 × 360 = 180 employees.

In summary, given the box-and-whisker plot, the number of employees that took a break that lasted between 37 - 41 minutes is: 180.

Learn more about box-and-whisker plot on:

https://brainly.com/question/3473797

Answer:

The answer is 90

Step-by-step explanation:

Its not 180 I take k12 also.

Select the correct answer.
Which number is an irrational number?

A.

B.

C.

D.

Answers

Answer:

its c i just got done doing the same one

Step-by-step explanation:

got it right on my real numbers test and got 100% if you need more help comment under my answer

Which represents the polynomial written in standard form? 4m – 2m4 – 6m2 9 9 4m 2m4 – 6m2 2m4 – 6m2 – 4m 9 9 – 6m2 4m – 2m4 –2m4 – 6m2 4m 9.

Answers

The standerd form 4m – 2m^4 – 6m^2 +9.

We have given that the polynomial given below we have to find the standerd form  of the polynomial.

What is the standerd form of the polynomial

[tex]P(x)=a_nx^n+a_{n-1}x^{n-1}+......+a_1x+a_0[/tex]

n=4

Standard form of the polynomial

[tex]4m - 2m^4 - 6m^2 + 9[/tex]

Rearranginge term we get

[tex]=-2m^4 - 6m^2 + 4m + 9[/tex]

The third term of the given polynomial is 0.

Therefore,the standard form is,

[tex]-2m^4 - 6m^2 + 4m + 9[/tex]

To learn more about polynomial visit:

https://brainly.com/question/5366408

i need help smh..anyways can someone help me? like asap-

Answers

Answer: x<-1

Step-by-step explanation:

The answer is A!

Answer:

See below.

Step-by-step explanation:

Solving the inequality :-

18 < -3(4x - 2)18 < -12x + 612 < -12x-1 > xx < -1

Graph A shows the solution to the inequality correctly

The amount of money you have after investing $400 for 8 years and $600 for 6 years at the same interest rate is represented by 400x^6 + 600x^6. where x is the growth factor. classify the polynomial by the number of terms. what is the degree?

Answers

The polynomial 400x⁶ + 600x⁶ has a degree of 6 and is a binomial.

What is a polynomial?

Polynomial is an expression that involves only the operations of addition, subtraction, multiplication of variables.

Polynomials are classified based on degree as linear, cubic, quadratic, etc.

A polynomial with two terms is known as a binomial. Hence:

The polynomial 400x⁶ + 600x⁶ has a degree of 6 and is a binomial.

Find out more on polynomial at: https://brainly.com/question/2833285

what type of sequence is 99,33,3,1

Answers

Answer:

36 ,36 30,2

Step-by-step explanation:

hope it's okay to you

please help I'll give brainliest if answer is right ​

Answers

Answer:

The area is about 96.06 or 96.1

Step-by-step explanation:

I tried my best to solve it, I'm sorry if it's wrong.

How you should answer:

1. Known:

2.Unknown:

3.Rationle:

4.Equation and Solution:


Steps on how to answer:

-State the known

-State the unknown and represent it with a variable

-Provide a rationale for your approach

-Include the equation, exact answer, and decimal approximation

Answers

Question 1

Focus on the triangle on the left.

sine is the ratio of opposite over hypotenuse

sin(angle) = opposite/hypotenuse

sin(44) = w/30

w = 30*sin(44)

w = 20.839751 approximately

Now move onto the triangle on the right. We'll use cosine this time.

cos(angle) = adjacent/hypotenuse

cos(x) = w/45

cos(x) = 20.839751/45

cos(x) = 0.463106

x = arccos(0.463106) ... arccosine is the same as [tex]\cos^{-1}[/tex]

x = 62.412286

Answers:w = 20.839751 approximatelyx = 62.412286 approximately

==========================================================

Question 2

Focus on the triangle on the left.

sin(angle) = opposite/hypotenuse

sin(32) = w/5

w = 5*sin(32)

w = 2.649596

Now move to the triangle on the right.

sin(angle) = opposite/hypotenuse

sin(x) = w/5

sin(x) = 2.649596/5

sin(x) = 0.5299192

x = arcsin(0.5299192) .... arcsine is the same as [tex]\sin^{-1}[/tex]

x = 31.999996

Answers:w = 2.649596 approximatelyx = 31.999996 approximately

==========================================================

Question 3

This time we use the tangent ratio.

Focus on the triangle that has legs of w and 16.

tan(angle) = opposite/adjacent

tan(70) = 16/w

w*tan(70) = 16

w = 16/tan(70)

w = 5.823524

Now let y = x+w and focus on the largest triangle this time.

tan(angle) = opposite/adjacent

tan(41) = 16/y

y = 16/tan(41)

y = 18.405895

This leads to:

y = x+w

x = y-w

x = 18.405895 - 5.823524

x = 12.582371

Answers:w = 5.823524 approximatelyx = 12.582371 approximately

what is the range of the exponential function y=2^x plus 2?



A. Y<0
B. Y<-1
C. Y<-2
D. Y<-1 (Less than or equal to)

Answers

It’s A
Explanation: because it’s A

sin 60 =11/x
x=11/sin (60)

Answers

Answer:

sixt

Step-by-step explanation:

thats easy the sin should be six

What is the slope of the line that passes through (-1, 3) and (-1, 8)?
due today

Answers

Answer:

Slope of the line is undefined

Step-by-step explanation:

(x₁, y₁) = (-1 , 3)  and (x₂, y₂) = (-1 , 8)

The line is parallel to y-axis. The slope of the line parallel to y-axis is undefined.

If X and Y are 2 finite sets such that n() = 9, n( XY) - 4 and n( XY) -15, then n() is
(A) 5
(B) 6
(C) 10
(D) 12​

Answers

Answer:

I think it's a hope this helps

Can someone please help me with this?

Answers

Answer:

50

Step-by-step explanation:

If we divide the shape we can get 3 x 10 with equals 30. Then take the remaining which is 2 x 10 and we get 20. Add the two to get 50. The area is 50sq miles.

Hope this helps. pls mark brainliest

Find the area of a circle whose radius is 3 1/2(three whole number one over two)

Answers

Answer:

38.5 in²

Step-by-step explanation:

Area of circle= πr², where r is the radius

Substitute the value of the radius into the formula above:

Area of circle

= π(3.5)²

= 12.25π

= 38.5 in² (3 s.f.)

Simplify: 5x3+3x2−(−x−6x2)+4x−x2

Answers

Answer: 33 plus 3x

Step-by-step explanation:  simplified it

Answer:

The answer is 5x3+8x2+5x

Step-by-step explanation:

Holding a ruler upright at arm’s distance (24 in.), Ronnie aligned the bottom of the ruler with a mark on the utility pole that was about 5 feet above the ground. He saw that the top of the pole aligned with the 6-inch mark on the ruler. Then he took 40 long strides to reach the pole. If each stride was about one yard (3 feet), then the top of the pole is about how many feet high?

Answers

Answer:

10 feet

Step-by-step explanation:

Drawing obviously not to scale but... Red segment is the ruler, at least the part between 5 and 6 inches, 1 inch long. Brown segment is the pole, ground to the top. Leftmost point is the eye, green line is the horizontal. The triangles are similar (AAA, the vertical lines are parallel), the ratio of the side is the same as the ratio of the heights. The height of the larger triangle (measured across the green line is

[tex]40 yd \times 3\frac{ft}{yd} \times 12\frac{in}{ft}= 1440''[/tex].

Ratio of the height is then [tex]1440\div24 = 60[/tex].

At this point the height of the pole is 60 times the length of the measure on the ruler, or 60 inches, that is 5 feet. Add the 5 feet the pole was starting from, it's 10 feet.

Find the volume of the square pyramid shown below.

Answers

We can use the formula: V = (1/3)a2h to solve this (a is not area, it’s for side length

We plug in our variables

V=(1/3)10^2x12

We get the answer 400m^3


find the value of x when y=14. y= -6x+5

Answers

Answer:

  x = -3/2

Step-by-step explanation:

To find the unknown value, substitute all known values and solve for the one remaining. After substitution, this becomes a 2-step linear equation, so is solved by the procedure for such equations.

  y = -6x +5 . . . . . given

  14 = -6x +5 . . . . substitute known value

  9 = -6x . . . . . . . subtract 5 (step 1)

  9/-6 = x = -3/2 . . . . divide by the coefficient of x (step 2);

Hello!

In order to find the value of x, we should take the value of y and plug it in and then simplify.

[tex]\bold{y=-6x+5}[/tex]

Plug in:

[tex]\bold{14=-6x+5}[/tex]

Switch places:

[tex]\bold{-6x+5=14}[/tex]

Now, subtract 5 from both sides:

[tex]\bold{-6x=14-5}[/tex]

[tex]\bold{-6x=9}[/tex]

Divide both sides by -6 in order to isolate x:

[tex]\bold{x=-\displaystyle\frac{9}{6}}[/tex]

Can this fraction be reduced? Sure, we can divide the numerator (9) and the denominator (6) by 3:

[tex]\bold{-\displaystyle\frac{3}{2}}[/tex]

As a Mixed Fraction:

[tex]\bold{1\displaystyle\frac{1}{2}}[/tex]

Hence, the value of x is:

[tex]\huge\boxed{\boxed{\mathfrak{\star{Answer:{\boxed{-\frac{3}{2} }}}}}}[/tex]

Hope everything is clear.

Let me know if you have any questions!

#KeepLearning

#NeverGiveUp

[tex]\rm{A~teen~who~never~gives~up}[/tex]

Find the perimeter.
Write your answer as a fraction or as a whole or mixed number.
3
1
4
in
112 in
1
in
12
NA
1
1
in
.
inches

Answers

Answer:

7 1/3 inches

Step-by-step explanation:

2 (1 3/4 + 1 11/12)

9+11

3/4+11/12= -------------

12

20/12

=1 2/3+ 2

2(3 2/3)

= 7 1/3

HELP ASAPPP
Inga, Kai, and Jason volunteer in the school library. They want to find the average number of pages of a library book. It would be unreasonable to calculate the number of pages in every book to find the average. They decide they need to find a valid sample.
Inga suggests choosing 100 books from the history section to get a sample.
Kai suggests randomly selecting 100 books from the library’s entire catalog.
Jason suggests using 100 books from the teen fiction section to get a sample.
Who suggested the best way to produce a random sample that is not biased?

Answers

Answer:

Kai suggested the best way to produce a random sample that is not biased

Step-by-step explanation:

Answer:

The answer is Kai suggested the best way to produce a random sample that is not biased

Step-by-step explanation:

Given cosθ = 3/5, find the five other trigonometric function values. 15

Answers

[tex]cos(\theta )=\cfrac{\stackrel{adjacent}{3}}{\underset{hypotenuse}{5}}\qquad \impliedby \textit{let's now find the \underline{opposite side}} \\\\\\ \textit{using the pythagorean theorem} \\\\ c^2=a^2+b^2\implies \sqrt{c^2-a^2}=b \qquad \begin{cases} c=hypotenuse\\ a=adjacent\\ b=opposite\\ \end{cases} \\\\\\ \sqrt{5^2-3^2}=b\implies \sqrt{25-9}=b\implies \sqrt{16}=b\implies 4=b \\\\[-0.35em] ~\dotfill[/tex]

[tex]sin(\theta )=\cfrac{\stackrel{opposite}{4}}{\underset{hypotenuse}{5}}\qquad \qquad tan(\theta )=\cfrac{\stackrel{opposite}{4}}{\underset{adjacent}{3}}\qquad \qquad cot=\cfrac{\stackrel{adjacent}{3}}{\underset{opposite}{4}} \\\\\\ sec(\theta )=\cfrac{\stackrel{hypotenuse}{5}}{\underset{adjacent}{3}}\qquad \qquad csc(\theta )=\cfrac{\stackrel{hypotenuse}{5}}{\underset{opposite}{4}}[/tex]

The values of the five other trigonometric functions value for the same input θ for which cos(θ) = 3/5 are:

sin(θ) =  4/5tan(θ) = 3/4cot(θ) = 4/3sec(θ) = 5/3csc(θ) = 5/4

What are the six trigonometric ratios?

Trigonometric ratios for a right angled triangle are from the perspective of a particular non-right angle.

In a right angled triangle, two such angles are there which are not right angled(not of 90 degrees).

The slant side is called hypotenuse.

From the considered angle, the side opposite to it is called perpendicular, and the remaining side will be called base.

From that angle (suppose its measure is θ),

[tex]\sin(\theta) = \dfrac{\text{Length of perpendicular}}{\text{Length of Hypotenuse}}\\\cos(\theta) = \dfrac{\text{Length of Base }}{\text{Length of Hypotenuse}}\\\\\tan(\theta) = \dfrac{\text{Length of perpendicular}}{\text{Length of base}}\\\\\cot(\theta) = \dfrac{\text{Length of base}}{\text{Length of perpendicular}}\\\\\sec(\theta) = \dfrac{\text{Length of Hypotenuse}}{\text{Length of base}}\\\\\csc(\theta) = \dfrac{\text{Length of Hypotenuse}}{\text{Length of perpendicular}}\\[/tex]

What is Pythagoras Theorem?

If ABC is a triangle with AC as the hypotenuse and angle B with 90 degrees then we have:

[tex]|AC|^2 = |AB|^2 + |BC|^2[/tex]

where |AB| = length of line segment AB. (AB and BC are rest of the two sides of that triangle ABC, AC being the hypotenuse).

We're given that:

cosθ = 3/5 for some angle θ

Since we've got:

[tex]\cos(\theta) = \dfrac{\text{Length of Base }}{\text{Length of Hypotenuse}}[/tex]

Therefore, we have:

[tex]\dfrac{3}{5} = \dfrac{\text{Length of Base }}{\text{Length of Hypotenuse}}[/tex]

Let we consider a right angled triangle in which there is hypotenuse of length 5 units and base (from the perspective of one of its non-right angle) of 3 units (as shown in the image attached below).

(we couldve taken 3x and 5x instead of 3 and 5, for any positive real number value of 'x' as when we would take their ratio, that common factor 'x' would get cancelled out. We can think of 3 and 5 as the special case of 3x and 5x when x = 1)

Then, from that perspective, let the perpendicular be of the length 'p' units, then as per the pythagoras theorem, we get:

[tex]p^2 + 3^2 = 5^2\\p = \sqrt{25 - 9} = \sqrt{16} = 4 \: \rm units[/tex]

(took only the positive root to remove the square term because the value of p denotes length, which is a non-negative quantity).

Thus, we have:

From the perspective of the angle θ:

Length of the base = 3 unitsLength of the perpendicular = 4 unitsLength of the hypotenuse = 5 units.

Thus, using these values, and the definition of the trigonometric ratios, we get:

sin(θ) =  4/5tan(θ) = 3/4cot(θ) = 4/3sec(θ) = 5/3csc(θ) = 5/4

Thus, the values of the five other trigonometric functions value for the same input θ for which cos(θ) = 3/5 are:

sin(θ) =  4/5tan(θ) = 3/4cot(θ) = 4/3sec(θ) = 5/3csc(θ) = 5/4

Learn more about Pythagoras theorem here:

https://brainly.com/question/12105522

Learn more about trigonometric ratios here:

https://brainly.com/question/22599614

#SPJ4

PLEASE HELP THANK YOU

Answers

Answer:

money left= -50•games played + 47.50

Hello people ~
what's the product of the square root of 3 and the square root of 12​

Answers

Answer:

6

Step-by-step explanation:

sqrt(3) x sqrt(12) = sqrt(3) x sqrt(4x3) = sqrt(3) x 2sqrt(3)

= 2 ( sqrt(3)² ) = 2 x 3 = 6

Let's see

[tex]\\ \rm\rightarrowtail \sqrt{3}\times \sqrt{12}[/tex]

[tex]\\ \rm\rightarrowtail \sqrt{3}\times \sqrt{2(2)(3)}[/tex]

[tex]\\ \rm\rightarrowtail \sqrt{3}\times 2\sqrt{3}[/tex]

[tex]\\ \rm\rightarrowtail 2(3)[/tex]

[tex]\\ \rm\rightarrowtail 6[/tex]

Use the equation, (1/27)^x=3^(-4x+6), to complete the following problems.
Rewrite the equation using the same base.
Solve for x. Write your answer as a fraction in simplest form.
Please show all work, and refrain from posting links, thank you!

Answers

Answer:

Given equation:

[tex]\left(\dfrac{1}{27}\right)^x=3^{(-4x+6)}[/tex]

27 can be written as [tex]3^3[/tex]

Also [tex]\dfrac{1}{a^b}[/tex] can be written as [tex]a^{-b}[/tex]

[tex]\implies \dfrac{1}{27}=\dfrac{1}{3^3}=3^{-3}[/tex]

Therefore, we can rewrite the given equation with base 3:

[tex]\implies (3^{-3})^x=3^{(-4x+6)}[/tex]

To solve, apply the exponent rule [tex](a^b)^c=a^{bc}[/tex]

[tex]\implies 3^{-3 \cdot x}=3^{(-4x+6)}[/tex]

[tex]\implies 3^{(-3x)}=3^{(-4x+6)}[/tex]

[tex]\textsf{If }a^{f(x)}=a^{g(x)}, \textsf{ then } f(x)=g(x)[/tex]

[tex]\implies -3x=-4x+6[/tex]

Add [tex]4x[/tex] to both sides:

[tex]\implies x=6[/tex]

A box of 100 personalized pencils costs $\$30$. How many dollars does it cost to buy 2500 pencils?

Answers

Answer:

$750

Step-by-step explanation:

2500 divided by 100 = 25 and 25 x 30 = 750

Sorry if im wrong

The price for 2500 pencils is $750.

What is Unitary Method?

The unitary technique involves first determining the value of a single unit,  followed by the value of the necessary number of units.

For example, Let's say Ram spends 36 Rs. for a dozen (12) bananas.

12 bananas will set you back 36 Rs. 1 banana costs 36 x 12 = 3 Rupees.

As a result, one banana costs three rupees. Let's say we need to calculate the price of 15 bananas.

This may be done as follows: 15 bananas cost 3 rupees each; 15 units cost 45 rupees.

We have,

A box of 100 personalized pencils costs $30.

So, cost of 1 pencil = $30 / 100 = 3/10

Now, the price to buy the 2500 pencils

= 2500 x 3/10

= 250 x 3

= $750

Learn more about Unitary Method here:

https://brainly.com/question/22056199

#SPJ2

Which number is a rational?

Answers

Answer:

0/5 = 0

0/200 = 0

0/ (-25) = 0

Step-by-step explanation: I am sure this is it because these are rational numbers

I need this for a geometry assignment........please help

Answers

All three are similar

In ∆SUT and ∆VWX

Two sides are similar and one angle is similar

So by SSA they are similar

In ∆PRQ and rest two

All angles are.

80,80,20°

So

They are similar by AAA congruence

i need help) Han and Clare go shopping, and they each have a coupon. Answer each question and show your reasoning.
1.Han buys an item with a normal price of $15, and uses a 10% off coupon. How much does he save by using the coupon?

2.Clare buys an item with a normal price of $24, but saves $6 by using a coupon. For what percentage off is this coupon?

Answers

The amount of money that Han saves by using a coupon is $1.5%

The percentage off Clare's purchase with the coupon is 25%

How much did Han save?

A coupon reduces the price at which an item is sold

Amount saved = coupon discouunt x normal price

10% x $15

0.1 x $15 = $1.5

What is the percent off Clare's coupon?

Percent off = (6/24 x 100  = 25%

To learn more about how to calculate discounts, please check: https://brainly.com/question/26061308

Other Questions
A student draws several cards from a deck of 10 unique cards.Which of these statements are true?Select all that apply.There are 252 different ways that 5 cards can be drawn.There are 120 different ways that 7 cards can be drawn.There are 210 different ways that 4 cards can be drawn.There are 240 different ways that 6 cards can be drawn.There are 90 different ways that 8 cards can be drawn. Mary and Jane went on a tour together. Mary brought twice as much money as Jane. After Mary had spent $800 and Jane had spent $240, they had equal amount of money left. How much money did Mary have at first? Where is the food for the tiny plant stored?embryoleafseed coatendosperm ___________ consists of a star and all of the bodies in orbit around it. How to do a dichotomous key =If f(x) is an exponential function where f(-2) = 25 and f(7) = 85, then find thevalue of f(14), to the nearest hundredth.=Exponential from two points Where is oxygen added to the blood? Directions: Answer each of the questions by typing your answer in the answer boxes.. If you get the answer correct, the box next to it will turn green. If you answer the question incorrectly, it will turn red. Coach West took his softball team to get pizza after the last game of the season. Their total bill was $128.50. If Coach West wants to leave the server a 22% tip, what is the TOTAL cost of the meal?Rosa and her sister went to a diner for lunch. They each ordered a salad for $7.20 and a soda for $1.50. If they left their server a 20% gratuity, what was the total cost of their meal?Mauricio and Frederick went to dinner at a seafood restaurant. The subtotal of their bill was $62.85. If they left their server a 22% gratuity, how much did the server receive?The Goodman family is dining at Cathy's Cafe. The subtotal of their meal is $84.50. If they left a total of $99.71 including tip, what percent of the subtotal did they leave as a gratuity?For his job, Keith earns 10% commission on his total sales. If Keith had a total sale of $655 last week, how much did he earn in commissions?For his job, Keith earns 10% commission on his total sales. To pay for a trip to Top Golf, Keith needs to earn $120 in commissions. What must Keiths sales total for the week?For her job, Carrie earns 20% commission on her total sales. If Carrie had a total sale of $440 last week, how much did she earn in commissions?For her job, Carrie earns 20% commission on her total sales. To pay for an upcoming vacation, Carrie needs to earn $200 in commissions. What must Carries sales total for the week? You are given the following data on different rates with the same maturity(1.5years) , but quoted on a different basis and different compounding frequencies : continuously compounded rate: 2.00% annualized rate continuously compounded return on maturity: 3.00% annually compounded rate: 2.10% annualized rate semi-annually compounded rate: 2.01% annualized rateyou want to nd an arbitrage opportunity among these rates.is there any one that seems to be mispriced? Abby made a quilt for her cousin doll The quilt had a 77 array of different colored square patches if each patch is 3 1/2 in long. What is the area of the whole quilt? Write an introductory paragraph on the topic: the benefits of e-learning Brainliest if correct 11Read this sentence from lines 11 and 12.It reclines in the middle of a quarry.The word reclines in this sentence most closely meansA lies downBstays alertClives foreverDturns to dust Could someone help me giving brainliest A man invests a sum of money ($8,000) in a bank that offers a rate of 5%. He invests the money for 2 years. Calculate the amount of money in the bank at the end of the 2-year period if invested at: (a) simple interest (b) compound interest. Question 6 (6 marks) The diameter of a circle is 10 cm (as shown in the diagram). Use = 3.14 Calculate to 2 decimal places, the: (a) circumference (b) area An angle measuring (870n) is in standard position. for which value of n will the terminal side fall on the y-axis? n = 3 n = 4 n = 5 n = 6 Pension expense must be adjusted when a net gain or a net loss at the beginning of the year exceeds 10% of the PBO or 10% of plan assets, whichever is higher. This threshold amount is referred to as the Help Please I need it!! anyone know how to answer questions helppp pleaseeeeeIn Noteflight, create an A chromatic scale, ascending and descending, using 8th notes in the treble clef.